HS 328 - Investments

Réussis tes devoirs et examens dès maintenant avec Quizwiz!

Which of the following can be greatly reduced by diversification? 1- Systematic risk 2-Market risk 3-Unsystematic risk 4-Systematic and unsystematic risk

3 - Unsystematic risk, is reduced by diversification.

Which of the following is most likely classified as a primary market transaction? 1. A mortgage-backed security (MBS) purchased from an institution that originates the mortgage loans 2. Equity shares purchased on the floor of the New York Stock Exchange 3. Corporate bond purchased from endowment fund that is liquidating 4. Shares of preferred stock that are purchased from a dealer

A: 1. Primary market transactions are those in which the financial securities are created, which would include the creation of an MBS by the originating institution. The equity and preferred shares and the corporate bond are examples of secondary market transactions.

One of your clients wants to build a portfolio characterized by high liquidity and very low risk. This client does not mind if the portfolio's returns are low. Given these specifications, you might recommend a portfolio consisting primarily of which asset class? A. Money market instruments B. Municipal bonds C. Corporate bonds D. Equities

A: A Money market instruments are the least risky asset class with the lowest return. Municipal and corporate bonds, as fixed income securities, have a moderate risk and return profile. Equities tend to offer the highest return as well as highest risk.

Most variation in portfolio returns over an extended period of time is attributable to: 1. security selection. 2. market timing. 3. asset allocation. 4. market cycles.

Answer: AA Over time, asset allocation represents the largest impact on performance for investors.

Jerry would like to purchase shares of a large, established company. He will most likely make his purchase: 1. in the primary market. 2. from an underwriter. 3. in the secondary market. 4. from the issuing corporation.

Newly issued shares are sold in the primary market, with the assistance of an underwriter. The purchase and sale of securities that have already passed through the primary market occurs in the secondary market. A: The shares of a large, established company have likely been through the primary market many years ago and are currently trading in the secondary market.

A small-cap fund manager invests in a technology company that grows and eventually becomes part of the S&P 500. What problem does the fund manager face by not selling shares of this company? A. Long-term capital gains B. Liquidity C. Style shift D. Low Jensen's alpha

The correct answer (C). When a small-cap fund successfully identifies firms that grow in value and become large firms, they need to sell shares of these firms to maintain their small-cap style orientation.

Which of the following advisors must register with the U.S. Securities and Exchange Commission (SEC)? 1. An advisor who sells insurance products 2. An advisor who is registered with the state securities regulator 3. An advisor who manages over $100 million in assets 4. An advisor who declines to be regulated by FINRA

3. An advisor who manages over $100 million in assets NOTE: Advisors with at least $100 million in assets under management must register with the SEC. Advisors with less than $100 million in assets under management must register with their respective state regulator.

All of the following statements are correct EXCEPT: 1. Yields on T-Bills are typically lower than commercial paper since there is less risk. 2. Negotiable CDs have very large balances and are tradeable in the secondary market. 3. Commercial paper is not considered default risk-free. 4. Interest income and capital gains from Treasury securities are both exempt from state and local taxes.

4. Interest income and capital gains from Treasury securities are both exempt from state and local taxes. The correct answer is (4). T-bills are default risk-free, while commercial paper has some, although minor, default risk. Interest income is exempt from state and local taxes, but capital gains are normally taxable at the state and local level.

All of the following is (are) among the responsibilities of FINRA EXCEPT: 1. educating investors. 2. fostering market transparency. 3. writing and enforcing rules governing the activities of all registered broker-dealer 4. firms and registered brokers in the U.S. writing and enforcing rules governing the activities of all registered investment advisors.

4. The SEC and state securities regulators are responsible for regulation of registered investment advisors.

A stock market participant that buys shares of stock in a secondary market and holds shares as part of its inventory is most likely a(n) A. stock exchange. B. independent advisor. C. broker-dealer. D. self-regulatory organization.

C. Broker dealers serve two functions: to act as an intermediary in helping clients buy and sell securities and to generate a profit on the trading of undervalued securities. In their dealer function, broker-dealers can hold inventory when they believe the shares are undervalued with the intent of selling them at a later date when the price rises.

An investor living in Pennsylvania who buys a municipal bond issued by the State of Wyoming will most likely 1. Pay federal taxes on the coupon payments. 2. Have less risk than with a Treasury security with a similar maturity date. 3. Earn a much higher yield to maturity than similar-risk corporate bonds. 4. Have a Pennsylvania tax liability on the interest received on the Wyoming bond.

Hide Feedback The correct answer is (4). Municipal bonds have tax advantages not enjoyed by corporate bonds, which makes their yields much less than those on corporate bonds. In addition, the interest received on municipal debt is federal tax-free and even state tax-free if the bondholders reside in the state of the issuer. In this case, the Pa. bondholder will have to report the income received on the Wyoming bond and will incur a state income tax liability.

Gene and Jean are married taxpayers who have a high income and are in the top marginal tax bracket. They own the home they live in. The home has greatly appreciated in value since he bought it and they are concerned about the taxes they will owe if they sell it. Which of the following statements best describe the tax-savings opportunities available to them? If they have lived in the home at least 2 of the past 5 years, they will not pay taxes on the first $500,000 of gain. They can depreciate the value of the home, saving on taxes today; and, when they sell the home, they can recapture the depreciation at a lower tax rate than the top marginal rate. If they perform a Section 1031 exchange, they may be able to defer or avoid taxation on the gain. If they gift the home to a relative, that relative will receive a step-up-in-basis; the relative can then sell the home without recognizing any gain.

Hide Feedback The correct answer is (A). As a married couple, Gene and Jean can exclude the first $500,000 of gain from their taxable income (assuming they lived in the home at least 2 of the past 5 years). Statements (B) and (C) are incorrect because these rules apply to business property but not personal property. Statement (D) is incorrect because the step-up-in-basis rules apply to inherited property but not gifted property.

A weakness of the buy-and-hold strategy is that it will almost certainly result in an asset allocation that looks much different than the original. returns far lower than what was expected. much higher investment expenses than other strategies. far more work on the behalf of the investment manager than other strategies.

Hide Feedback The correct answer is (A). Buy and hold implies that the investor does not rebalance the portfolio weights even after significant swings in prices. This can result in weights that are substantially different from the beginning-of-period weights and very likely to produce a portfolio with greater volatility than originally intended.

Colin purchased Apple stock earlier this year. The stock has increased significantly and Colin believes that it has reached its upper range in value. He would like to sell but wants to avoid the gain being taxed as a short-term capital gain. What is his best option? Buy Apple puts. Sell Apple puts. Buy Apple calls. Sell Apple calls.

Hide Feedback The correct answer is (A).A put option protects against drops in Apple stock price.

Which of the following regarding futures contracts is least accurate? Futures contracts are less liquid than forward contracts. Futures contracts are marked to market. Futures contracts are traded on a regulated exchange. Futures contracts allow more delivery options than forward contracts. .

Hide Feedback The correct answer is (A).Futures contracts are more liquid than forward contracts, because they are traded on organized exchanges and are standardized. Forward contracts are customized to the parties involved in the transaction

Index funds are most notable for which one of the following features? Low management fees Outperformance of the market Liquidation during open-market hours Trading on exchanges

Hide Feedback The correct answer is (A).Index fund managers do not perform significant levels of research on security selection and allocation, which tends to reduce management fees. Index funds are among the funds with the lowest expenses and fees. Option (B) is incorrect because index funds do not tend to outperform the market; rather, they tend to mimic the market. Option (C) is incorrect because ETFs, not index funds, may be liquidated during open-market hours. Option (D) is incorrect because closed-ended funds and ETFs are traded on exchanges; index funds are bought and sold directly from the fund manager.

Which of the following is a violation of the wash-sale rule? You sell 27 shares of an S&P 500 index fund at a loss. Then 15 days later, you buy 42 shares of a Russell 3000 index fund. You have brokerage accounts at two firms. At the first firm, you buy 100 shares of PQR stock. Then 22 days later, you sell 100 shares of PQR stock, bought the prior year, for a loss in your account at the other firm. Your spouse buys 200 shares of UK stock. Then 31 days later, you sell 100 shares of the same stock, bought 43 days earlier, at a slight loss. You sell 100 shares of UK stock at a slight gain. Then 4 days later, your spouse buys 200 shares of the same stock. Hide Feedback The correct answer is (B).Option (A) is incorrect because the two indices are sufficiently different. Option (C) is incorrect because the purchase is more than 30 days prior to the sale. Option (D) is incorrect because the stock is sold at a gain.

Hide Feedback The correct answer is (A).Isabella gave loss property to her daughter. When a taxpayer transfers property with a loss to someone else, and the recipient sells the property for an amount between the donor's adjusted basis and the fair market value of the stock on the date of the gift, no gain or loss is recognized. In this example, the no-gain, no-loss corridor is from $10,000 (the fair market value of the stock on the date of the gift) to $15,000 (the adjusted basis in the hands of the donor). If Shea sold the stock for any amount between $10 and $15 thousand dollars, she is not required to recognize any gain, and she is prohibited from recognizing any loss on the transaction. Since there is no gain or loss, there is no need to categorize the tax result as either long or short term.

The process of mark-to-market posts gains or losses to each account daily and may result in margin calls. impacts only short positions. occurs only on the contract's expiration date. is not required on all futures contracts.

Hide Feedback The correct answer is (A).Mark-to-market efficiently puts futures contracts on a "pay-as-you-go" basis. It is required for all contracts.

Which type of mutual fund share generally converts to an "A share" after a period of time? Class B shares Class C shares Both B and C shares Neither B nor C shares

Hide Feedback The correct answer is (A).Most Class B shares have a conversion feature that allows the shares to convert to Class A after a specified time period. Class C shares do not convert to A shares.

Which of the following statements concerning a unit investment trust is correct? It has an unmanaged portfolio. It usually consists of common stocks. It is most appropriate for young people. It requires constant management attention. Hide Feedback The correct answer is (A).Option (B) is incorrect as UITs generally consist of bonds. Option (C) is incorrect as UITs are better for generating income, not the typical objective of young people. Option (D) is incorrect as the fund is generally passive.

Hide Feedback The correct answer is (A).Option (B) is incorrect as UITs generally consist of bonds. Option (C) is incorrect as UITs are better for generating income, not the typical objective of young people. Option (D) is incorrect as the fund is generally passive.

Primary commodities likely include: Oil extracted from wells Refined gasoline Steel produced from iron ore "2 by 4s" made in a lumber yard

Hide Feedback The correct answer is (A).Primary commodities are those that are considered natural resources that can be mined or extracted and used without much processing needed for consumption. Examples include oil, gold, and wheat. When a commodity must undergo a significant process to be able to be consumed, it is known as a secondary commodity. Examples include refined gas, lumber, and steel.

The North Equity Fund has a beta of 1.67 and a standard deviation of 22.6 percent. It has returned 13.8 percent during the past year when the return on one-year treasury bills has been 3.6 percent. The Sharpe Ratio of the North Equity Fund is closest to 0.45 0.61 6.11 8.26

Hide Feedback The correct answer is (A).Sharpe Ratio = (Portfolio Return - Risk-Free Rate) ÷ (Standard Deviation)Sharpe Ratio = (13.8 − 3.6) ÷ 22.6 = 0.45

An investor buys a call option on ABC stock with an exercise price of $45 for $5 per share. When ABC increases to $50, the call is exercised, and the stock is immediately sold. What is the result? A loss of $500 No gain or loss A profit of $500 A profit of $1,000

Hide Feedback The correct answer is (B). ($5 gain from stock price × 100) − ($5 premium × 100) = $0

A corn farmer who wants to hedge the price of corn should enter into what type of contract? Buy a corn futures contract. Sell a corn futures contract. Long position in a corn call-option contract. Long position in a commodity put-option index.

Hide Feedback The correct answer is (B).A corn farmer would enter into a short (sell) futures contract, because he wants to sell his corn.

Philip buys undeveloped beachfront property. He is subject to all of the following risks EXCEPT Market risk Default risk Liquidity risk Political risk

Hide Feedback The correct answer is (B).Philip, as an investor in real estate, is subject to market risk, liquidity risk and political risk. He is not subject to default risk. There is no indication that there is debt involved in this question. In addition, he would not be the one subject to default risk even if debt was involved in the purchase of the real estate.

Which of the following statements concerning convertible bonds is (are) correct? I. Convertible bonds provide an inexpensive way to buy stock.II. Convertible bonds provide a lower yield than do straight bonds of comparable credit risk and maturity because of the attractiveness of the conversion feature. I only II only Both I and II Neither I nor II

Hide Feedback The correct answer is (B).Statement I is incorrect because convertible bonds normally trade at a premium-to-conversion value. Thus, buying a convertible bond and then converting the bond provides an investor with fewer shares of stock than if the investor had purchased the stock outright directly at the start.

Higgins purchased 2,000 shares of Dunlap, Inc., stock for $28 per share three years ago and paid a $50 brokerage commission on the transaction. This year he sold the shares for $37 per share and paid a $50 brokerage commission on the transaction. What is the amount of his taxable gain or loss? $0 $17,900 $18,000 $74,000

Hide Feedback The correct answer is (B).The amount realized from the sale is $74,000 − $50 commission = $73,950. His cost basis is $56,000 + $50 commission = $56,050. The taxable gain is $73,950 − $56,050 = $17,900.

Three years ago, Ty purchased 1,000 shares of Pope Industries, Inc., for $10 per share. He signed an agreement with the company that allowed the company to use his dividend payments to purchase additional shares for him. Over the last 3 years, Ty received a total of $1,200 in dividend payments, which purchased an additional 100 shares of stock. If Ty sells all of his shares for $24,000, what is his taxable gain? $0 $12,800 $14,000 $24,000

Hide Feedback The correct answer is (B).Ty's adjusted basis in the shares equals the initial purchase price of the shares, $10,000, plus the dividends of $1,200 that were reinvested, since Ty was required to pay tax on the dividend payments for the year in which they were made. When he sells the shares, he realizes $24,000 less his adjusted basis of $11,200, which equals a taxable gain of $12,800.

Based on Malkiel's theorems, which of the following statements is correct? I. In terms of percent change, longer maturity bonds are less volatile than shorter term bonds when interest rates fluctuate.II. In terms of percent change, higher coupon bonds are less volatile than lower coupon bonds when interest rates fluctuate. I only II only Both I and II Neither I nor II

Hide Feedback The correct answer is (B).When interest rates change, bonds with higher coupon payments and shorter maturities are less volatile.

An individual seeking diversification by including alternative investments in her portfolio is most likely to benefit if there are tax advantages associated with the alternative investment. the alternative investment generates high returns. there are low correlations between her portfolio and the alternative investment. the alternative investment is a good hedge against inflation.

Hide Feedback The correct answer is (C).Diversification only works if there is less than a perfect +1.0 correlation between a portfolio and the alternative investment. The primary reason for the popularity of alternatives is that they tend to have low or even negative correlations with traditional securities.

Private equity is regularly characterized by the ability to accurately estimate the beta of a target business. low funding risk. performance that is affected by economic conditions. active secondary markets to trade private equity positions.

Hide Feedback The correct answer is (C).It is very difficult for private equity firms to gain enough access to relevant financial information of target businesses to be able to accurately estimate the target's beta. PE has significant funding risk at least some of the time, and there is a limited secondary market. The overall performance of the PE firm depends significantly on the state of the economy.

Which of the following investing risks is most significant for real estate investors as compared to investors in publicly traded stocks? Market risk Default risk Liquidity risk Idiosyncratic risk

Hide Feedback The correct answer is (C).Liquidity risk is far higher in real estate investing than in trading in stock that can be sold on a secondary market exchange. Stock investors are subject to market risk, idiosyncratic risk, and the risk that the firm will declare bankruptcy.

Which of the following is an advantage of mutual funds over ETFs? Lower stock turnover Lower average expenses Ability to redeem at net-asset value Greater tax efficiency

Hide Feedback The correct answer is (C).Mutual fund shares can be redeemed for net-asset value while ETFs can be sold at market value, which will be very close to net-asset value if the shares are highly liquid. ETFs generally are more tax efficient because they tend to have lower stock turnover, and on average, have lower expenses.

Consider a European call option and a European put option, written on the same underlying stock. Both options have the same strike price and the same maturity date. What does the put-call parity imply about the price of these two options? I. Independent of the value of the underlying asset, both options must have exactly the same price before and at maturity.II. At the maturity date, the price of the call option minus the price of the put option equals the price of the underlying stock minus the strike price.III. Before the maturity date, the price of the call option minus the price of the put option equals the current price of the underlying stock minus the appropriately discounted strike price.IV. The price of both options is always equal to the current value of the underlying asset. I and II I and IV II and III II and IV

Hide Feedback The correct answer is (C).Statements II and III are correct. These statements reflect the put-call parity model.

Mutual fund 12b-1 fees cover which of the following? Portfolio management fees Administrative costs Advertising and marketing fees Organizational costs

Hide Feedback The correct answer is (C).The 12b-1 fee structure allows for up to 1% allowance for distribution costs, including advertising and promotion.

Jacques has a convertible bond with a par value of $1,000 that is trading in the market for $925. The bond is convertible into 50 shares of XYZ stock. The current market price of XYZ stock is $17.50 per share. What is the bond's conversion premium? $0 $76 $50 $125

Hide Feedback The correct answer is (C).The bond is convertible into 50 shares, which would be worth $875. The bond is trading at $925. Thus, the conversion premium equals the difference, or $50.

The following terms of commodity futures contracts are specified by the exchange. Answer Choice Quality Quantity Delivery Date Maximum Price Change Over the Term of the Contract (A) (B) X X (C) X (D) X A B C D

Hide Feedback The correct answer is (C).The contract specifies a daily price limit, which restricts price movement in a single day.

A venture capitalist provides EUR 25 million to a business that has developed an innovative fishing product. The product has been successfully tested in the Mediterranean Sea but needs some adjustments and a packaging model. Distribution channels have not yet been identified, although some revenues have been generated. The VC has most likely entered the process at the seed stage. emerging stage. startup stage. expansion stage.

Hide Feedback The correct answer is (C).The startup stage is characterized by businesses that have a product to sell but need help with marketing, packaging, and distribution. The VC frequently acts as an expert in this stage by providing advice and consulting services. The startup stage can produce some revenues, but the business lacks mass production capabilities.

An "in the money put" has an intrinsic value < 0. is where the stock price > strike price. is where the stock price < strike price. has a time value > 0.

Hide Feedback The correct answer is (C).The stock price is less than the strike price.

All of the following are correct regarding derivative securities EXCEPT: Derivatives can be used for speculation and hedging. The use of leverage is both an advantage and a disadvantage of derivatives. The value of a derivative is tied to the value of an underlying security or asset. Most types of derivatives are suitable investments for unsophisticated investors.

Hide Feedback The correct answer is (D). The use of leverage exaggerates both gains and losses, which can be an advantage or a disadvantage. This exaggeration of gains and losses, along with a derivative's relative complexity, makes most types of derivatives unsuitable for small, unsophisticated investors.

A contract that is an agreement for immediate exchange of funds for assets is called a futures contract. forward contract. call option contract. spot contract.

Hide Feedback The correct answer is (D).A spot contract is a contract that calls for immediate delivery of goods. The spot price or rate is the current price or rate.

Debra entered into a "forward contract" to buy 100 shares of Petra stock at a price of $15 per share in exactly 9 months. Now, the nine-month period has expired. Petra stock is trading at $18 per share. What are the economic consequences for Debra? She has lost $800. She has gained $1,100. She has lost $300. She has gained $300.

Hide Feedback The correct answer is (D).Debra has gained $300. ($18 − $5) × 100 shares.

All of the following positions are examples of the types of holdings found in a hedge fund manager's portfolio EXCEPT the long-only position in a gasoline futures contract. a put option written on a barrel of oil. long and short positions in the equity of two industry leaders. the long position only in a highly rated municipal bond.

Hide Feedback The correct answer is (D).Hedge funds rarely invest in investment grade municipal bonds, because the risk is generally very low and the return is not sufficient to warrant interest by the hedge fund manager. Hedge funds often invest in options and futures contracts and also try to benefit when one industry leader outperforms its main rival.

Cody purchased 400 shares of NAY stock six years ago when it was trading at $65 per share. Unfortunately, NAY has been steadily declining. Cody sold his shares this year for $18 per share. This year Cody also sold 800 shares of a mutual fund that he purchased 6 months ago. His average cost per share was $15 and he sold the shares for $32. Assuming Cody had no other capital transactions this year, what is Cody's tax consequence from these transactions? $5,200 long-term loss this year $18,800 long-term loss, $13,600 short-term gain taxed at ordinary income rates this year $3,000 ordinary loss this year with a $15,800 ordinary loss carried forward up to 5 years $3,000 ordinary loss this year with a $2,200 loss carried forward indefinitely .

Hide Feedback The correct answer is (D).The NAY stock sale resulted in a long-term loss of $18,800 ($47 loss per share × 400 shares). The mutual fund sale resulted in a short-term gain of $13,600 ($17 gain × 800 shares). The entire $13,600 gain is offset by the loss, leaving a net long-term loss of $5,200. Cody can take $3,000 of the loss against ordinary income this year, and the remaining loss is carried forward indefinitely (until death)

Identify the financial asset with the highest Treynor ratio if the risk-free rate of interest is 3%. Asset Average Return Standard Deviation Beta R2 Fund 1 10% 13% 0.7 70 Fund 2 18% 22% 1.2 65 Fund 3 20% 30% 1.7 85 Index A 14% 17% 1.0 100 Fund 1 Fund 2 Fund 3 Index A Hide Feedback The correct answer is (B). Fund 1 (10 − 3) ÷ 0.70 = 10 Fund 2 (18 − 3) ÷ 1.2 = 12.5 Fund 3 (20 − 3) ÷ 1.7 = 10 Index A (14 - 3) ÷ 1.0 = 11

Hide Feedback The correct answer is (D).Unlike corporate bonds, whose coupon payments are taxed as ordinary interest income, the coupon payments from municipal bonds are not subject to tax under the regular tax system and are not subject to the NIIT. If the taxpayer is subject to AMT, the coupon payments from private activity municipal bonds are taxable; however, the coupon payments from public purpose municipal bonds remain tax-free. Gains from the sale of municipal bonds are taxable gains.

Tommy compares the fees and expenses of several equity funds. Tommy will likely have the most difficulty in evaluating 1. trading costs. 2. the 12b-1 fees. 3. the back-end loads. 4. the front-end loads.

The correct answer is (1). Trading costs are the most difficult for an investor to assess. Other fees and expenses are disclosed to investors in the prospectus, but also because some trading costs are implicit. Examples of implicit trading costs are bid-ask spreads, market-impact costs, and opportunity costs.

Which of the following choices correctly orders Treasuries, corporate bonds, and equities from the least chance of a loss of principal to the most chance of a loss of principal? 1. Treasuries, corporate bonds, equities 2. Treasuries, equities, corporate bonds 3. Equities, corporate bonds, Treasuries 4. Corporate bonds, Treasuries, equities

The correct answer is (1).Corporate bonds have higher default risk than Treasuries, but they offer better principal protection than equities.

Which of the following statements regarding collateralized mortgage obligations (CMOs) is correct? 1. All tranches receive principal payments throughout the term. 2. All tranches receive interest payments throughout the term. 3. Tranches w/ shorter maturities are not subject to prepayment risk. 4. Tranches with longer maturities are not subject to default risk.

The correct answer is (2). While interest is paid to all tranches, principal is paid only to the first tranche until that tranche is retired, and then it is paid to the second tranche until it is retired, and so on. All tranches are subject to default risk and prepayment risk.

All of the following regarding TIPS are true EXCEPT: 1. The interest paid on TIPS changes based on inflation and the changing principal amount of the TIPS. 2. TIPS can be purchased on a competitive or noncompetitive basis. 3. TIPS have maturities that do not exceed 20 years. 4. If deflation occurs over the life of the TIPS, the investor receives the greater of the principal amount or the par value.

The correct answer is (3). TIPS are issued by the U.S. Treasury on both a competitive and noncompetitive basis. The interest rate does not change. Instead, the principal amount changes based on inflation or deflation. Maturities for TIPS can be as long as 30 years. Upon maturity, investors receive the greater of par value or principal value.

Consider this chart with the seven portfolios and the efficient frontier. H B C D (efficient frontier) F A G All of the following statements are correct EXCEPT: 1- Portfolio H does not exist in the real world. 2- Portfolio C dominates portfolios F and G. 3-Portfolios B, C, and D are all efficient and are optimal portfolios for an investor. 4-Portfolio D is the riskiest option.

The correct answer is (3). Option (3) is NOT correct; while it is true that these portfolios lie on the efficient frontier, the optimal portfolio for any given investor is one that both lies on the frontier and is appropriate for the investor's risk capacity and risk tolerance. The other statements are correct. No portfolio may exist above the efficient frontier, so Portfolio H is unattainable. Portfolio C dominates F and G because it offers more return for the same level of risk. Portfolio D is the riskiest option because it lies furthest to the right.

All of the following statements regarding U.S. Treasury securities are correct EXCEPT: 1. Treasury securities are issued in an auction format. 2. It is possible to buy a fraction of a Treasury bond because the minimum investment is only $100. 3. Individual investors CANNOT purchase Treasury issues directly from the Treasury Department. 4. The same yield is paid by all investors.

The correct answer is (3).Individuals can make purchases directly from the Treasury Department via the Treasury Direct website. The yield on a specific issue is set by the lowest competitive bid, which then applies to all competitive and noncompetitive bids.

The return from the CAPM is which of the following? 1. The risk-free return 2. The actual return 3. The risk-premium 4. The expected return

The correct answer is (4). CAPM = Rf + β(Rm - Rf ) = The Expected Return based on beta, the market premium and the risk-free rate of return.

The intersection of the Security Market Line (SML) and the y axis occurs at the: 1. market premium. 2. market portfolio. 3. real rate of return. 4. risk-free rate of return.

The correct answer is (4). The SML is a line that connects: Risk-free rate of return (y-axis), to the market portfolio.

All of the following are likely to be classified as investment advisers by the U.S. Securities and Exchange Commission EXCEPT 1. a financial analyst who issues a report on the performance of value stocks. 2. an analyst who recommends a triple-A-rated bond to a client. 3. a money manager who makes asset allocation assignments for institutional investors. 4. a financial firm that acts as a dealer in investment grade bonds.

The correct answer is (4). Broker-dealers are explicitly excluded from being classified as advisers under the legal framework of the Investment Advisers Act. Individuals or firms that issue reports or offer investment advice, including asset-allocation decisions, are considered to fit the definition of an investment adviser.

A portfolio has the following stocks: Portfolio Weight Expected Return Stock 1 50% 20% Stock 2 20% 7% Stock 3 30% 6% What is the weighted return of the portfolio? 13.2% 16.0% 14.0% 10.0%

The correct answer is (A). (0.50 × 20) + (0.20 × 7) + (0.30 × 6) = 13.2

Which of the following statements about mutual fund loads is correct? A back end load may be charged when an investor redeems shares of a mutual fund shortly after buying them. A no-load mutual fund will have either no front-end load or no back-end load. Loads are considered transaction costs and do not appear in the fund's prospectus. Back-end loads are included in a fund's annual expense ratio until the load is paid off.

The correct answer is (A). Back-end loads are typically charged when an investor redeems shares of a mutual fund shortly after buying those shares. Statement (B) is incorrect because a no-load fund will have neither a front-end nor a back-end load. Statement (C) is incorrect because loads are not considered transaction costs and will appear in a fund's prospectus. Statement (D) is incorrect because back-end loads are not included in the expense ratio and will never be paid unless the investor redeems shares.

A financial analyst computes the present value of a firm's operating cash flows to calculate an intrinsic value of the shares. The analyst is most likely using A. the discounted cash-flow technique. B. the relative valuation methodology. C. indexing. D. technical analysis.

The correct answer is (A). Discounted cash-flow analysis includes estimating future cash flows and then discounting them at an appropriate interest rate. The present value is often referred to as the intrinsic value of the company.

Characteristics of preferred stock that indicate it is a hybrid security most likely include: 1. Representation of ownership interest 2. Guaranteed dividend payments 3. Premiums attached to its par value 4.Payment of interest

The correct answer is (A). Preferred stock represents an equity ownership position. Preferred shareholders typically receive a fixed dividend (not interest) payment. No dividend payments are ever guaranteed. Par value is independent of the notion of a hybrid security.

Examples of income stocks are least likely to include stock categories: Tech Blue chip Utility Financial institutions

The correct answer is (A). Tech stocks typically are high growth firms that require any cash flows to be reinvested back into the business, leaving little for the payment of dividends. Blue chips, utilities, and financial institutions generally pay dividends, often even high dividends.

Which of the following is accurate regarding the capital market line (CML)? 1. Some combination of the risk-free asset and the market portfolio will always be preferred to a portfolio of N risky assets (excluding the market portfolio itself). 2. The CML is a line that begins at the risk-free rate of return and crosses the efficient frontier at the market portfolio. 3. The CML represents the most efficient portfolios, consisting of various percentages of risky assets. 4. The CML consists of combinations of the risk-free return and the market portfolio with the slope equal to standard deviation.

The correct answer is (A). The CML is a combination of the risk-free return and the market portfolio. The CML does not cross the efficient portfolio, rather it is tangent to it. The CML consists of the market portfolio and the Rf. The incorrect as the slope is not equal to the standard deviation.

When interest rates increase, which of the following strategies might be helpful in order to take advantage of the decrease in the value of fixed-income securities? The tax swap The spread swap The bullet strategy The barbell strategy

The correct answer is (A).As interest rates increase, the price of bonds decreases. This drop in value may create an opportunity to recognize the capital loss for tax purposes, which is the purpose of the tax swap.

Brooks has been following Amazon stock for years. All forms of analysis leads him to firmly believe the stock is overpriced. Based on this conclusion, he shorts 1,000 shares of Amazon. Which of the following positions would help hedge his short? Buy 10 Amazon call options. Sell 10 Amazon call options. Buy 10 Amazon put options. Sell 10 Amazon put options.

The correct answer is (A).Brooks has a short stock position, which means he borrowed the shares that he sold short. If the stock price increases, his loss potential is unlimited. The long call allows the Amazon stock to be bought at a fixed price, limiting his risk.

Juan bought XYZ Company stock at $40 per share. Today, the stock sells for $52 per share. Juan likes the long-term prospects for XYZ stock but wants some protection against price decreases. Which of the following orders is the best way for Juan to both participate in future price increases and ensure a minimum profit of $6 per share? A. A limit order to sell at $46 B. A stop-limit order, stop price = $47, limit price = $46 C. A limit order to buy at $40 D. A stop-sell order, stop price = $46

The correct answer is (B). A limit order will not work as it is a sell order at or above $46 that would be executed today. It does not provide the downside protection Juan is looking for. The stop-limit order will place a limit order with a price of $46 once the stock reaches $47. The stop-sell order results in a market order once the stock price reaches $46, which likely will result in the order being filled below $46

Devon Baines is a board member with DDO, Inc., a large pharmaceutical firm. At a recent meeting, the board votes to take over a smaller competitor whose shares trade in the OTC market and are priced at $50 per share. DDO will offer $75 per share, and the offer will become public 10 hours after the meeting. Baines decides to purchase shares in the target firm immediately after the meeting, but when he calls his broker to place the order, he is informed that the price of the target firm is $72. The market is most likely in a(n) A. weak form. B. strong form. C. semi-strong form. D. inefficient form.

The correct answer is (B). A market in which prices reflect all relevant information, including private information, is a strong-form market. In this case, Baines was hoping to purchase at $50 but was only able to place the order when the price had risen to $72 per share, indicating that the price already reflected the new private information about the takeover. This can only occur in a strong-form market.

National Fund is a closed-end fund w/ a cash balance of $80M. American Fund is an open-end fund w/ a cash balance of $300M. Both National and American have $2B in market value of assets. National has less cash most likely because: 1. it invests in less liquid assets than American. 2. does not permit investors to redeem shares at any time. 3. has greater management fees than American. 4. has less access to capital than American.

The correct answer is (B). Open-end funds must have much more cash on hand than closed-end funds because their shareholders can redeem shares daily, and those transactions occur at the closing net asset value of the fund. Closed-end fund shareholders must sell their shares on an exchange to redeem them, thus requiring closed-end funds to have minimal cash on hand.

Portfolio A has a weighted beta coefficient of 1.5, and Portfolio B has a weighted beta coefficient of 0.9. With these assumptions, which of the following statements is correct? A. Because Portfolio A has a beta greater than 1.2, it would be preferred by rational investors. B. Assuming the market were to drop by 5 percent, Portfolio B should drop less than Portfolio A. C. Neither portfolio is as volatile as the market. D. Portfolio A would be considered a "defensive" portfolio.

The correct answer is (B). Option (A) is incorrect because there is insufficient information to know whether a rational investor would prefer Portfolio A; we would also need to know the portfolio's return to make this judgement. A positive alpha, on the other hand, would be preferred by rational investors. Options (C) and (D) are incorrect, as Portfolio A is more volatile than the market.

A large firm announces that it has hired a design company to change its corporate logo. The new logo will be shown publicly for the first time the following day. Shareholders react by bidding up the shares by 10 percent over the relevant benchmark. No other information about this firm is released. The shareholder reaction can best be described as: A. anchoring. B. representativeness. C. cognitive dissonance. D. mental accounting. .

The correct answer is (B). Representativeness is a simple heuristic decision-making tool in which investors make quick decisions without regard to probability distributions or searching for the true factors that should influence the decision. When investors bid up the price of a company's shares just because it changes its logo, the shareholders are hoping the new logo will increase the firm's profitability. Since no other information is released around this time, the price reaction is likely attributed to representativeness

Which of the following comments would most likely be made by an investor influenced by behavioral finance rather than modern portfolio theory? A. My risk aversion should guide my asset allocation. B. I'd rather not add international stocks to my portfolio because they lost money last year. C. I should sell some of my stock allocation because they went up in value last quarter. D. It probably doesn't make sense to try to pick which stocks will outperform in the future.

The correct answer is (B). Stocks will randomly rise and fall, and international stocks should be part of any well-diversified portfolio. Failing to invest in international stock because of a recent loss means that the investor is likely exhibiting loss aversion. The other three statements are consistent with modern portfolio theory. Asset allocation should be guided by risk aversion, rebalancing means selling assets that have risen in value, and efficient markets suggest little value to stock picking.

Which of the following is an index designed to track the performance of large- and mid-cap securities from 21 developed countries? A. Wilshire 21 B. MSCI EAFE C. Barron's 400 D. Euro 21

The correct answer is (B). The MSCI EAFE index is often used as a benchmark for foreign asset managers. The MSCI EAFE measures' equity's outside the US & Canada. The index includes a selection of stocks from 21 developed markets.

Which rebalancing strategy incurs the lowest transaction costs? A. The constant-weighting strategy B. The buy-and-hold strategy C. The tactical allocation strategy D. The dynamic allocation strategy

The correct answer is (B). The buy-and-hold strategy is a strategy that avoids the transaction costs of periodic rebalancing by simply allowing investments to rise or fall in value over time. The primary disadvantage of this strategy is that one's asset allocation will be affected by the past performance of investments held in the portfolio. In order to preserve a desired asset allocation, some other form of rebalancing may be preferable despite the higher transaction costs.

Onion, a large technology company, has stock that pays a $1 dividend semiannually. Its current earnings per share is $10. If the stock is currently trading at $120, what is the dividend yield percentage? 1. 0.83% 2. 1.67% 3. 10.00% 4. 20.00%

The correct answer is (B). The dividend yield percentage is equal to the total annual dividends per share divided by the stock price. ($1 × 2) / $120 = 1.67% Formula: Dividend Yield Percentage: Amount of $Dividend x frequency paid year / $ Share Value

Fred bought 100 shares of Apple at $115 per share. One year later, he sold the stock for $152 per share. During the year, Apple declared and paid dividends of $2 per share. What was Fred's holding period return? 1. 32% 2. 34% 3. 73% 4. 206%

The correct answer is (B). (Net proceeds + dividend - interest) ÷ equity invested = HPR [($152 - $115) + $2] ÷ $115 = 33.91%

Richie has been saving diligently for many years and has accumulated an appropriate emergency fund along with substantial retirement savings that will allow her to enjoy all of her favorite activities during retirement and still make generous gifts to her nieces and nephews over the next several years. Richie experiences extreme anxiety when his portfolio declines in value. Which of the following is true? Richie's willingness (tolerance) to take on risk is greater than his ability (capacity) to take on risk, so her portfolio should have a moderately aggressive asset allocation. Richie's ability (capacity) to take on risk is greater than his willingness (tolerance) to take on risk, so her portfolio should have a conservative asset allocation. Richie's willingness (tolerance) to take on risk is greater than his ability (capacity) to take on risk, so her portfolio should have a conservative asset allocation. Richie's ability (capacity) to take on risk is greater than her willingness (tolerance) to take on risk, so her portfolio should have a moderately aggressive asset allocation.

The correct answer is (B). Risk capacity is a measurement of the amount of risk a client can afford to take on. Risk tolerance is the amount of risk a client is willing to take on. When risk capacity and risk tolerance are not in alignment, the more conservative of the two should determine the planning recommendations. In this case, Richie has the ability to take on risk; however, since he lacks the tolerance (willingness) to take on risk, her portfolio should be invested with a more conservative asset allocation.

Security A has the following returns over 4 years: 4%, 7%, 0%, and -1%. What is the mean return and the standard deviation (sample) for Security A? A. Mean of 2.5% and standard deviation of 3.2% B. Mean of 2.5% and standard deviation of 3.7% C. Mean of 4% and standard deviation of 3.2% D. Mean of 4% and standard deviation of 3.7%

The correct answer is (B). The mean is 2.5 percent and the standard deviation is close to 3.7 percent. You can calculate these using the Σ+ key on a financial calculator.

The Anderson bond is a 5 percent coupon bond with semiannual coupon payments that matures in 10 years. If the YTM for this bond is 4 percent, what is the value of the bond? $1,081.11 $1,081.76 $1,124.35 $1,125.03

The correct answer is (B).P/YR 2 FV $1,000.00 N 20 i 4.0% Pmt $25.00 PV $1,081.76

Bryce purchased 1,000 shares of Canadian Red Tree Company (CRT), which had a price of CAD 81 exactly 6 months ago. The price of CRT has subsequently declined to CAD 60. Spring has stated that he will not sell his shares until the price reaches at least CAD 81. Spring most likely shows signs of: A. framing. B. anchoring. C. mental accounting. D. cognitive dissonance.

The correct answer is (B).Spring is clearly anchoring his decision on the original price at which he inherited the shares instead of making an informed decision based on the future profitability of CRT. Anchoring tends to force investors to hold on to losing stocks.

Which of the following bonds are most likely to be CALLED? 1. Zero-coupon bonds 2. Coupon bonds selling at a discount 3. Coupon bonds selling at a premium 4. Floating-rate bonds

The correct answer is (C). Bonds sell at a premium when prevailing interest rates are lower than the rate on the existing bonds. Since new rates are lower, the issuing corporation will find it desirable to call the existing bonds and issue new bonds at the lower rates. The interest rate on floating-rate bonds will adjust periodically to changes in prevailing interest rates.

Marleen, who lives in Louisiana, is in the 20 percent federal tax bracket and 5 percent state income tax bracket. Which of the following bonds that she is considering purchasing has the highest after-tax yield? A treasury bond paying 4% A corporate bond paying 4.5% A Louisiana municipal bond paying 3.5% A Texas municipal bond paying 3.6%

The correct answer is (C). Corporate bonds are subject to federal and state income tax. Treasury bonds are subject to federal income tax only. Municipal bonds are not subject to federal income tax, but they are subject to state income tax if they are not issued by the taxpayer's state of residence. The La. muni has the highest after-tax return. Treasury = 3.200% Corporate = 3.375% La. Bond = 3.500% Texas Bond = 3.42%

Cashflow, Inc. expects to generate $200 million in operating cash flows during the next year. It estimates its long-term dividend growth rate to be 3 percent, and it has 100 million shares outstanding. What is the intrinsic value of Cashflow, Inc., if your required rate of return is 9 percent? A. $16.67 B. $22.22 C. $33.33 D. $66.67

The correct answer is (C). Intrinsic value = Total market value ÷ Outstanding Shares Total market value = Operating Cash Flow ÷ (Required Rate of Return - Dividend Growth Rate) Total market value = $200,000,000 ÷ (0.09 − 0.03) = $200,000,000 ÷ 0.06 = $3,333,333,333 Intrinsic value = $3,333,333,333 ÷ 100,000,000 = $33.33

Mike inherits 600 shares of Apple stock from his Aunt. He finds that when he reads stories about Apple he tends to spend more time on positive articles that highlight the company's potential for growth while dismissing articles that suggest that a lack of new products will negatively impact growth. Mike may be exhibiting: A Heard mentality. B Gambler's fallacy. C Cognitive Dissonance. D Anchoring.

The correct answer is (C). Mike's tendency to pay attention to information consistent with his beliefs is compatible with cognitive dissonance.

Which of the following techniques or strategies would take advantage of a perceived undervaluation in the energy sector of the economy? 1- Dollar-cost averaging into a portfolio 2- Strategic asset allocation rebalancing 3- Tactical asset allocation 4- Index funds

The correct answer is (C). Tactical asset allocation is in response to short-term market conditions. Dollar-cost averaging is a method of investing over time. Strategic asset allocation is a long-term strategy, not one that takes advantage of sector imbalances.

If an investor owns a single share of a stock with a beta of 0.75, what can you conclude about his investment risk relative to the market? 1. The investor's total risk is 3/4 of the risk of the market. 2. The investor is taking 75 percent more total risk than the risk of the market. 3. The investor's systematic risk is ³⁄₄ of the risk of the market. 4. The investor's diversifiable risk is 75% more than the total risk of the market.

The correct answer is (C). Beta only measures systematic risk relative to 1, the market's beta. Beta assumes a stock is added to an already diversified portfolio. Because a single stock has both diversifiable as well as systematic risk, an investor who purchases only a single stock may have a total risk greater than the market's risk.

Andy is considering purchasing a 12-year bond that is selling for $1,300. What is the yield to maturity (YTM) for this bond if it has an 8 percent coupon, paid semiannually? a. 4.63% b. 4.68% c. 4.70% d. 4.72%

The correct answer is (C). P/YR: 2 PV: ($1,300) N: 24 Pmt: $40 FV: $1,000 i: 4.70%

Sam's retirement fund is expected to earn a nominal rate of 7 percent, and the inflation rate is estimated at 3 percent. What is Sam's real rate of return? 1.43% 2.33% 3.88% 4.00%

The correct answer is (C). Real return = (1.07 ÷ 1.03) − 1 = 3.8835%

Sam has a $3-million fixed-income portfolio that consists of Bond A, Bond B, Bond C, and Bond D. The bonds have durations of 2, 3, 8, and 10, respectively. If Sam has 20 percent invested in Bond A, 30 percent in Bond B, and 25 percent invested in each of the other two bonds, what is the duration for the portfolio? Assume that the correlation among the bonds is 0.5. 5.50 5.75 5.80 6.20

The correct answer is (C).(0.2 × 2) + (0.3 × 3) + (0.25 × 8) + (0.25 × 10) = 0.4 + 0.9 + 2.0 + 2.5 = 5.8

Andrea believes that the economic cycle is at its peak and that a severe downturn is likely in the near future. She would like to invest in a stock that will remain relatively stable if the expected downturn does take place. Which of the following is the most suitable investment to meet her goal? A growth stock A stock with a beta close to 1 A defensive stock A stock with a beta higher than 1

The correct answer is (C).Defensive stocks are firms that provide products that are necessary for everyday life and are relatively unaffected by general fluctuations in the economy. They typically have betas of less than one.

Byte, Inc., had net earnings of $2.5 billion last year. It has 500 million shares of common stock outstanding, which stayed the same since last year. It paid 50 cents per share per quarter this year as a dividend payment, which is the same as last year. This year's earnings are 20% higher than last year. Which of the following is correct for this year? The retention ratio equals 33.33%. The retention ratio equals 50.00%. The payout ratio equals 33.33%. The payout ratio equals 40.00%.

The correct answer is (C).The dividend per share equals $2.00. The EPS equals $5.00 last year, which is found by dividing net earnings by outstanding shares. This year's EPS is 20% higher or $6.00.The payout ratio = dividend / EPS $2.00 / $6.00 = 33.33%

Stock purchases with a 50% initial margin and a 30% maintenance margin will be called at a 15% decrease in the price of the stock. 25% decrease in the price of the stock. 28.6% decrease in the price of the stock. 20% decrease in the price of the stock.

The correct answer is (C).The formula used to determine the price below when a margin call will occur is: debt / (1 − MMR). For example, if a 50% initial margin rate and a 30% maintenance margin rate apply, a stock that was purchased when the price was $100 would receive a margin call if the price falls below $50 / (1 − 0.30) = $71.42. The price drop is $28.58, or 1 − debt / (1 − MMR).= 1 − (0.50 / 0.70) = 28.58%.

Jerry invested in a mutual fund 5 years ago. His returns were 25%, -5%, 10%, 0%, and 50%, respectively. What is the difference between the arithmetic average and the geometric average return over the 5 years? 1 - 0% 2 - 1.1% 3- 1.3% 4- 1.6%

The correct answer is (D). Arithmetic average = 16% Geometric average = [(1 + 25%) × (1 + -5%) × (1 + 10%) × (1 + 0%) × (1 + 50%)](1/ 5) - 1 = 14.4% Difference = 16% − 14.4% = 1.6%

Which of the following statements correctly identifies significant differences between modern portfolio theory (MPT) and behavioral finance (BF)? A. MPT assumes all investors have incomplete knowledge of investments, while BF assumes investors work with perfect knowledge. B. MPT assumes investors are averse to losses, while BF assumes investors are averse to risk. C. MPT assumes investors price securities heuristically, while BF assumes investors price securities rationally. D. MPT assumes new information is quickly priced into the market, while BF assumes that price adjustments are not immediate and are incomplete.

The correct answer is (D). MPT assumes efficient markets and rational investors. Answers (A), (B), and (C) reverse the assumptions between the two theories.

Which is the LEAST likely motivation for investing in mutual funds? A. To seek diversification benefits B. To obtain professional management of investments C. To invest conveniently D. To obtain returns that normally exceed market returns

The correct answer is (D). Mutual funds offer diversification, convenience, and professional management Mutual funds frequently perform well below the market return.

A financial advisor decides to eliminate presenting quarterly changes in portfolio balance and instead decides to present progress toward the client's long-term goals. Avoiding a client's tendency to respond emotionally to negative performance is associated with: gambler's fallacy. anchoring. representativeness. prospect theory.

The correct answer is (D). Prospect theory suggests that investors respond more strongly to a loss from a reference point than economic theory would predict. Gambler's fallacy is the tendency to believe that a random event is "due" for a gain after experiencing a loss. Anchoring is the tendency to fixate on the original value when making relative decisions. Representativeness is the tendency to assess new information using superficial traits rather than actual probabilities.

Maggie considers four potential securities with identical expected returns but different correlations with her existing portfolio as part of her asset-selection decision. Which security would provide Maggie with the greatest diversification benefit? Security 1 with a correlation coefficient of +1.0 Security 2 with a correlation coefficient of +0.32 Security 3 with a correlation coefficient of 0.00 Security 4 with a correlation coefficient of -0.38

The correct answer is (D). Since the returns of the four potential securities are the same, the primary consideration is the risk element. The best security is the one with the lowest correlation, which is Security 4.

Parker, who lives in Covington, Louisiana, purchased three bonds from a company based in Brazil that were yielding 9.75 percent and paid a 12 percent coupon semi-annually. The company went bankrupt, and Parker never received his money. What type of risk was he subject to when he purchased the bond? A. Interest-rate risk B. Default risk C. Exchange-rate risk D. All of the above

The correct answer is (D). The Brazilian bond is subject to exchange-rate risk, default risk, and interest-rate risk.

Assume that the 1-, 2-, 3-, 5-, 10-, 20-, and 30-year rates were 7 percent, 6.5 percent, 6 percent, 5.4 percent, 5.2 percent, 5.0 percent, and 4.8 percent, respectively. What type of yield curve is this? A. Humped B. Normal C. Flat D. Inverted

The correct answer is (D). The yield curve is downward sloping, which is known as inverted.

Tom is considering two portfolios: 1) Portfolio A with a return of 10 percent and a standard deviation of 20 percent, and 2) Portfolio B with a return of 6 percent and a standard deviation of 8 percent. Assuming the correlation between A and B is 0.2 Tom invests 40 percent in A 60 percent in B What is the portfolio standard deviation?

The correct answer is (D). Standard deviation:

An asset-allocation strategy that periodically purchases assets that have fallen in value and sells assets that have risen in value is a buy-and-hold strategy. a tactical allocation strategy. a dynamic allocation strategy. a constant-weighting strategy.

The correct answer is (D).A constant-weighting strategy returns the weights to their original strategic values. Consequently, when the price of a security falls, its weights also fall. The investor is forced to purchase additional shares to increase the allocation.


Ensembles d'études connexes

Anatomy Lecture Final Term 2 (quizzes)

View Set

Training Leaders in a Diverse Multicultural Environment Mid-Term Study Notes

View Set

Chapter 12 - Intervention Planning

View Set

SOC101 - Module 5 - Week 2 - Health and Aging

View Set

Human Anatomy and Physiology Chapter 1

View Set

Pregnancy: Conception, Fetal Development, & Maternal Adaptation

View Set

MS EXAM 2 EMPHASIZED INFO (Jeopardy, Quizzes, PP Questions, Evolve, Critical Thinking)

View Set